Block sliding on vertical track with friction (Solved problem)

  • Thread starter kuruman
  • Start date
  • Tags
    Physics
  • #1
kuruman
Science Advisor
Homework Helper
Insights Author
Gold Member
2023 Award
14,359
7,838
Homework Statement
A small block of mass ##m## is sliding on the inside of a vertical circular track of radius R. The coefficient of kinetic friction between the block and the track is μ. The block starts at the 9 o'clock position (point A) with speed ##v_0## and reaches the 3 o'clock position (point B) along two separate paths: (a) clockwise (top semicircle) or (b) counterclockwise (bottom semicircle). Assume that initial speed is high enough to keep the block in contact ith the track at all times. Gravity acts conventionally from top (12 o'clock) to bottom (6 o'clock).

Find the final speed at point B of the mass along the to paths.
Relevant Equations
##F_{net} = ma.##
This problem is a reformulation and solution of a problem posted earlier that did not go very far. A streamlined solution is posted here for future reference.

Solution
Shown below are two FBDs for the two paths.

TwoArcsWithFriction.png


From the top FBD,
##N+mg\cos\theta=m\omega^2 R\implies N=m\omega^2 R-mg\cos\theta##
From the bottom FBD,
##N-mg\cos\theta=m\omega^2 R\implies N=m\omega^2 R+mg\cos\theta##
We combine the two into a single equation and write
##N=m\omega^2 R\mp mg\cos\theta.##
The force of friction for the two cases is
##f_k=\mu N=\mu m(\omega^2 R\mp g\cos\theta).##
The top/bottom sign corresponds to the top/bottom FBD. Before writing the tangential acceleration, we note that, in each case, ##~-\frac{\pi}{2}\leq \theta \leq \frac{\pi}{2}## Since the particle starts at A and ends at B in each case, ##\sin\theta## is negative in quadrants II (top FBD) and quadrant III (bottom FBD). The positive direction in each case is the direction of the linear velocity. Thus, the linear acceleration is
Top FBD:##~a_{\text{top}}=-\dfrac{f_k}{m}+g\sin\theta##
Bottom FBD:##~a_{\text{bot}}=-\dfrac{f_k}{m}-g\sin\theta##
The tangential acceleration for two cases is $$a_t=-\frac{f_k}{m}\pm g\sin\theta=-\mu(\omega^2 R\mp g\cos\theta)\pm g\sin\theta=-\mu\omega^2 R \pm g(\sin\theta-\mu\cos\theta).$$Now $$a_t= R\frac{d\omega}{dt}=R\frac{d\omega}{d\theta}\frac{d\theta}{dt}=\omega R\frac{d\omega}{d\theta}.$$This results in the differential equation $$\omega \frac{d\omega}{d\theta}=-\mu\omega^2 \pm \frac{g}{R}(\sin\theta-\mu\cos\theta)$$ which we have to solve in order to answer the question. In the absence of gravity (##g=0##), the equation becomes ##~\dfrac{d\omega}{d\theta}=-\mu\omega~## which has solution ##\omega = Ce^{-\mu~\theta}.## This motivates trying a solution $$\omega(\theta)=f(\theta)e^{-\mu~\theta}.$$ The left-hand side is $$LHS=\omega \frac{d\omega}{d\theta}=fe^{-\mu~\theta}\left(\frac{df}{d\theta}e^{-\mu~\theta}-\mu fe^{-\mu~\theta}\right)=f\frac{df}{d\theta}e^{-2\mu~\theta}-\mu\omega^2.$$ Setting this equal to the RHS and canceling what cancels yields a separable equation, $$f\frac{df}{d\theta}=\pm \frac{g}{R}(\sin\theta-\mu\cos\theta)e^{2\mu~\theta} \implies f~df=\pm g(\sin\theta-\mu\cos\theta)e^{2\mu~\theta}d\theta.$$ Using Wolfram Alpha for the integral on the RHS, we obtain $$
\begin{align} & \frac{1}{2}\left(f^2-f_0^2\right)=\mp\frac{g}{R}\frac{(2\mu^2\cos\theta-\mu\sin\theta+\cos\theta)e^{2\mu \theta}}{4\mu^2+1} \nonumber \\
& f=\left [f_0^2 \mp \frac{2g}{R} \frac{(2\mu^2\cos\theta-\mu\sin\theta+\cos\theta)e^{2\mu \theta}}
{4\mu^2+1} \right]^{1/2} \nonumber \\
& \implies \omega= \left [f_0^2 \mp \frac{2g}{R} \frac{(2\mu^2\cos\theta -\mu\sin\theta+\cos\theta)e^{2\mu \theta}}{4\mu^2+1} \right]^{1/2}e^{-\mu~\theta} \nonumber \\
\end{align}$$ The initial condition is ##\omega(-\pi/2)=v_0/R## and can be used to find ##f_0##. $$ \begin{align}
& \frac{v_0}{R}=\left [ f_0^2 \mp\frac{2g}{R}\frac{\mu e^{-\mu \pi}}{(4\mu^2+1)} \right]^{1/2}e^{\mu \pi/2}. \nonumber \\
& f_0^2=\left[ \frac{v_0^2}{R^2}\pm \frac{2g}{R}\frac{\mu}{(4\mu^2+1)}\right]e^{-\mu \pi}. \nonumber \\
\end{align}$$ Finally, $$\omega= \left \{\left[ \frac{v_0^2}{R^2}\pm \frac{2g}{R}\frac{\mu}{(4\mu^2+1)}\right]e^{-\mu \pi} \mp \frac{2g}{R} \frac{(2\mu^2\cos\theta -\mu\sin\theta+\cos\theta)e^{2\mu \theta}}{4\mu^2+1} \right\}^{1/2}e^{-\mu~\theta}.$$ We test this expression by setting ##\mu =0## whch is the frictionless case. $$\begin{align}
& \omega= \left( \frac{v_0^2}{R^2} \mp \frac{2g}{R}\cos\theta\right)^{1/2} \nonumber \\
& \frac{v^2}{R^2}=\frac{v_0^2}{R^2} \mp \frac{2g}{R}\cos\theta \nonumber \\
& \frac{1}{2}mv^2=\frac{1}{2}mv_0^2 \mp mgR\cos\theta \nonumber \\
& \frac{1}{2}mv_0^2=\frac{1}{2}mv^2 \pm mgR\cos\theta. \nonumber \\
\end{align}$$The last equation is the familiar statement of mechanical energy conservation for the top and bottom paths with the potential energy being zero at the horizontal diameter.

At last we are in a position to find the speed at point B. $$\begin{align}
&\omega_B= \left \{\left[ \frac{v_0^2}{R^2}\pm \frac{2g}{R}\frac{\mu}{(4\mu^2+1)}\right]e^{-\mu \pi} \mp \frac{2g}{R} \frac{( -\mu)e^{\mu \pi}}{(4\mu^2+1)} \right\}^{1/2}e^{-\mu~\pi/2} \nonumber \\
& \omega_B= \left[\frac{v_0^2}{R^2}e^{-2\mu\pi}\pm\frac{2\mu g}{(4\mu^2+1)R}(1+e^{-2\mu\pi}) \right]^{1/2}\nonumber \\
& \implies v_B=\left[ v_0^2e^{-2\mu\pi}\pm \frac{2\mu gR}{(4\mu^2+1)}(1+e^{-2\mu\pi})\right]^{1/2} . \nonumber \\
\end{align}$$
Shown below is a plot of ##v(\theta)=\omega(\theta)~R## for the two paths with plotting parameters as shown.

TwoArcsSpeed.png
 
Last edited:
  • Like
  • Informative
Likes Juanda, anuttarasammyak, MatinSAR and 1 other person
Physics news on Phys.org
  • #2
One quick question. I find it odd the first expression you use lacks a ##-## sign. It might just be related to how you defined the positive directions of your coordinates but I wanted to confirm it because in the original thread, I already had doubts about the sign convention (post #35) and I still couldn't fully solve them.

You wrote:
kuruman said:
From the top FBD,
##N+mg\cos\theta=m\omega^2 R##

However, if positive ##r## grows outwards and positive ##\theta## grows counterclockwise, then it'd have a minus sign. Also, I'm having trouble to see how ##N## and ##mg\cos\theta## could be positive as well. They point inwards making ##r## smaller so it feels like they'd be negative. Similar to how in a xy coordinate system, a left-pointing force is negative when it accelerates the body in the negative direction.
This page shows the derivation for the formulas. It's unfortunate it uses ##\phi## instead of ##\theta## and also it's got some missing equations due to formatting errors but the main idea is still there.
1698258550646.png


Can you add how you defined ##+r## and ##+\theta##? I assume ##+r## is pointing outwards and I'm guessing ##+\theta## is pointing clockwise instead of counterclockwise which might explain why you didn't get that ##-## sign.
 
  • #3
Juanda said:
One quick question. I find it odd the first expression you use lacks a ##-## sign. It might just be related to how you defined the positive directions of your coordinates but I wanted to confirm it because in the original thread, I already had doubts about the sign convention (post #35) and I still couldn't fully solve them.

You wrote:However, if positive ##r## grows outwards and positive ##\theta## grows counterclockwise, then it'd have a minus sign. Also, I'm having trouble to see how ##N## and ##mg\cos\theta## could be positive as well. They point inwards making ##r## smaller so it feels like they'd be negative. Similar to how in a xy coordinate system, a left-pointing force is negative when it accelerates the body in the negative direction.
Strictly speaking you are correct. If you look at the top FBD, the radial components of ##\vec N## and ##m\vec g## are negative and so is the centripetal acceleration. This means that the equation should have been written conventionally as $$-N-mg\cos\theta=-m\omega^2 R.$$However, if you multiply both sides of the equation by ##-1##, you get all the negative signs converted to positive signs which is what I have. What matters is the relative signs between terms, not the overall sign.
Juanda said:
This page shows the derivation for the formulas. It's unfortunate it uses ##\phi## instead of ##\theta## and also it's got some missing equations due to formatting errors but the main idea is still there.
View attachment 334232
I don't disagree with this page and there is no misfortune to using ##\phi## instead of ##\theta##. You can see how these more general forms become what I have if you set ##\phi=\theta##, ##\dot \phi=\omega## and ##r=R=\rm{constant}##:
##F_r=-m\omega^2R##
##F_{\theta}=mR\dfrac{d\omega}{dt}.##

Juanda said:
Can you add how you defined ##+r## and ##+\theta##? I assume ##+r## is pointing outwards and I'm guessing ##+\theta## is pointing clockwise instead of counterclockwise which might explain why you didn't get that ##-## sign.
Angle ##\theta## is shown in the drawing of the two FBDs.
For the top diagram, ##\theta## increases clockwise as the mass moves from point A to point B. At A ##\theta =-\frac{\pi}{2}## and at B ##\theta =+\frac{\pi}{2}.##
For the bottom diagram, ##\theta## increases counterclockwise as the mass moves from point A to point B. At A ##\theta =-\frac{\pi}{2}## and at B ##\theta =+\frac{\pi}{2}.##
 
  • Informative
Likes Juanda

1. How do you calculate the acceleration of a block sliding on a vertical track with friction?

The acceleration of a block sliding on a vertical track with friction can be calculated using the equation a = (m*g*sinθ - μ*m*g*cosθ)/m, where m is the mass of the block, g is the acceleration due to gravity, θ is the angle of the track, and μ is the coefficient of friction.

2. What is the coefficient of friction and how does it affect the block's motion on the track?

The coefficient of friction is a measure of the amount of friction between two surfaces. In this case, it represents the friction between the block and the track. A higher coefficient of friction means there is more resistance to motion, so the block will have a slower acceleration and take longer to reach the bottom of the track.

3. How does the angle of the track affect the block's motion?

The angle of the track, θ, directly affects the acceleration of the block. As the angle increases, the component of gravity pulling the block down the track decreases, resulting in a slower acceleration. At a certain angle, the block will not slide at all and will remain stationary on the track.

4. Can the block ever reach the bottom of the track?

Yes, the block can reach the bottom of the track if the angle of the track is steep enough and the coefficient of friction is low enough. However, if the angle is too shallow or the coefficient of friction is too high, the block may not have enough acceleration to reach the bottom and will stop at some point on the track.

5. How does the mass of the block affect its motion on the track?

The mass of the block, m, affects its motion on the track by changing its inertia. A heavier block will have a greater resistance to motion, so it will have a slower acceleration and take longer to reach the bottom of the track compared to a lighter block. However, the mass does not affect the angle or coefficient of friction of the track.

Similar threads

  • Introductory Physics Homework Help
Replies
6
Views
570
  • Introductory Physics Homework Help
Replies
3
Views
223
  • Introductory Physics Homework Help
Replies
10
Views
267
  • Introductory Physics Homework Help
Replies
6
Views
2K
  • Introductory Physics Homework Help
Replies
14
Views
2K
  • Introductory Physics Homework Help
Replies
17
Views
402
  • Introductory Physics Homework Help
Replies
5
Views
474
  • Introductory Physics Homework Help
Replies
7
Views
218
Replies
8
Views
239
  • Introductory Physics Homework Help
Replies
1
Views
355
Back
Top